LSAT and Law School Admissions Forum

Get expert LSAT preparation and law school admissions advice from PowerScore Test Preparation.

 Administrator
PowerScore Staff
  • PowerScore Staff
  • Posts: 8916
  • Joined: Feb 02, 2011
|
#72969
Complete Question Explanation

Parallel Flaw. The correct answer choice is (D).

The last question of the section presents an interesting mash-up of both conditional and causal reasoning, wherein one condition (immunity to a microorganism) is sufficient to prove that such a microorganism will not cause harmful symptoms (an anti-causal claim inside a necessary condition). The stimulus then proceeds essentially like any Mistaken Reversal, wherein the author asserts that because a microorganism did not cause harmful symptoms (the neccessary condition occurred), that is sufficient to prove immunity to that microorganism (the sufficient condition occurred.) In this Parallel Flaw question we need to find an answer that matches that flaw - a Mistaken Reversal, preferably with a causal or anti-causal claim embedded in one of the conditions.

Answer choice (A): While this answer is flawed, it is not due to a Mistaken Reversal, but an incorrect application of Formal; Logic, and it also lacks the causal element we are looking for. (Note that this answer would have been valid if the author had said some things that best serve the interests of everyone are not morally right, and reversing that claim as this answer did is not, technically speaking, a Mistaken Reversal of conditional reasoning, but is due to the incorrect application of the concept of "some.")

Answer choice (B): Although some of the claims in this answer could be converted to conditional claims, there is no Mistaken Reversal present, but more like a Mistaken Negation. Also, "probably" in the conclusion is a very different strength than what we saw in the stimulus. Loser.

Answer choice (C): This answer is solely causal, with no conditional elements to it, and is therefore a complete mismatch for our stimulus.

Answer choice (D): This is the correct answer choice. Here we have a Mistaken Reversal, and some implied causal reasoning in one of the conditions (with excessive taxation as the cause and less expansion as the effect). This is a match, and therefore our correct choice.

Answer choice (E): Nothing conditional to see here, just a claim about what happens less often within one group than in others, and a conclusion that there is no cause and effect involved. Absent the overriding conditional framework for the argument, the causal part is not enough to match the stimulus and this answer must be rejected.
 carnegie49
  • Posts: 19
  • Joined: Apr 12, 2016
|
#25325
Could someone please setup the conditional diagram for the stimulus?

Thanks!!
 David Boyle
PowerScore Staff
  • PowerScore Staff
  • Posts: 836
  • Joined: Jun 07, 2013
|
#25412
carnegie49 wrote:Could someone please setup the conditional diagram for the stimulus?

Thanks!!

Hello carnegie49,

There is a Mistaken Reversal problem here, more or less. The stimulus can be diagrammed as, or something close to,

immunity :arrow: harmful symptoms
harmful symptoms :arrow: immunity

, which chimes with the similar bad logic in answer D.

Hope this helps,
David
 taylorballou
  • Posts: 18
  • Joined: Feb 18, 2017
|
#41250
Hello,

Could someone please confirm what the diagram is for answer B?

I got:

Advertisers --> try to persuade people
Advertisers-- > try to persuade people

Now that I'm reviewing the answer choices again, I feel like "writers of fiction" doesn't necessarily fall into the "advertisers" category?

Thank you,

Taylor
 Luke Haqq
PowerScore Staff
  • PowerScore Staff
  • Posts: 739
  • Joined: Apr 26, 2012
|
#41969
Hi Taylor!

The diagrams you provided for (B) look good to me.

Even though this is a Flaw in the Reasoning question and (B) does contain flawed reasoning, it doesn't parallel the flawed reasoning in the stimulus.

The flaw in the stimulus is that it fails to take into consideration an alternative cause. People exposed to staphylococcus without developing any harmful symptoms might not develop symptoms for reasons other than immunity to infection, and the argument fails to consider this possibility. Answer (D) similarly fails to consider an alternative cause. In (D), business owners might be less willing to expand their businesses for reasons other than excessive taxation.

Hope that helps!
 fersian
  • Posts: 19
  • Joined: Jan 19, 2019
|
#65001
Hello.
I am just trying to get some clarity on why 'A' is wrong.
Is it because it is formal logic, and not necessarily conditional reasoning?
Could a formal logic answer choice be a correct answer for a parallel flaw question when the stimulus is conditional? Or does it have to mimic the same structure as well as the flaw?

Could you check the diagramming of answer 'A'?
I know it is wrong, but I want to get some clarity for the future.

MR - Morally right
J - Just
SI- serve interests of everyone

MR --all--J
SI ---> ~J
MR --s-- ~SI
 Brook Miscoski
PowerScore Staff
  • PowerScore Staff
  • Posts: 418
  • Joined: Sep 13, 2018
|
#65017
Fersian,

I would not make the distinction that (A) lacks conditional reasoning, although you on a correct track. It has some conditional reasoning and some other formal logic. The stimulus has a word "many" that could be associated with formal logic (and not conditional reasoning) as well.

The best way to distinguish is, as you suggest, to diagram.

Stimulus:
Premise: I :arrow: -H
Conclusion: -H :arrow: I (qualified by a many--but the conclusion is restrained to just those people)

This is a Mistaken Reversal, and the formal logic word doesn't really play a role.

(A):
Premise 1: M :arrow: J
Premise 2: -J :some: E
The Flaw (unstated): The argument incorrectly interprets "some" as "not all," thus assuming J :some: -E
Conclusion: M :some: -E

(A) has multiple/more complex premises, the "formal logic" word actually plays a role, and the flaw is indeed not a conditional flaw but a formal logic flaw--failing to realize that "some" includes the possibility of "all."
User avatar
 smtq123
  • Posts: 29
  • Joined: May 28, 2021
|
#91109
In option A, Why the below interpretation is wrong? Kindly explain
Premise 2: E (Some) ------> -J
Contrapositive of Premise 2: J (Some) -----> -E

Many thanks for your support .
 Adam Tyson
PowerScore Staff
  • PowerScore Staff
  • Posts: 5153
  • Joined: Apr 14, 2011
|
#91532
The problem with that diagram, smtq123, is that a Formal Logic statement based on a "some" claim does not have a valid contrapositive. It's not conditional, and we cannot treat it the same way as we treat conditional statements. Here's an example to illustrate:

Some New Yorkers don't live in Albany:

NY :some: Albany

(Notice that "some" is actually a double arrow - if some New Yorkers do not live in Albany, it must also be true that some people who do not live in Albany are New Yorkers)

Now, can we reverse that and get a valid statement that means "some people who live in Albany are not New Yorkers"? Nope - everyone who lives there is also in New York!

Formal Logic looks a lot like Conditional Reasoning, but different rules apply and different inferences can be drawn. Don't mix those two up! For more on Formal Logic, check out the modules in the Online Student Center (if you are a Powerscore course or tutoring student) or in the Logical Reasoning Bible (if you are doing self-study).

Get the most out of your LSAT Prep Plus subscription.

Analyze and track your performance with our Testing and Analytics Package.